Zur Asymptotik wechselwirkender Korrelationsfunktionen

Betrachten Sie eine interagierende QFT (z. B. im Zusammenhang mit den Wightman-Axiomen ). Lassen G 2 ( X ) sei die Zweipunktfunktion eines Körpers ϕ ( X ) :

G 2 ( X ) = ϕ ( X ) ϕ ( 0 )

Frage : Was ist über das Verhalten von bekannt G 2 1 ( P ) bei P ? Gibt es eine Begrenzung für seine Wachstumsrate?

Es wäre schön, ein (nicht störungsfreies) Theorem für den allgemeinen Spin zu haben, aber falls dies nicht möglich ist, können Sie davon ausgehen ϕ ( X ) ist skalar. Auch jeder Hinweis ist willkommen.

Einige Beispiele:

Ein freies Skalarfeld hat

G 2 1 ( P ) = P 2 + Ö ( 1 )
während ein wechselwirkender, nach erster Ordnung in der Störungstheorie, hat
G 2 1 ( P ) = C P 2 + Ö ( Protokoll P 2 )
für einige C > 0 . Natürlich gibt es in der Störungstheorie große Logarithmen aller Ordnungen, und daher repräsentiert dieses Ergebnis nicht das Wahre P Verhalten von G 2 ( P ) . Man könnte im Prinzip die führenden Protokolle zu allen Ordnungen summieren, aber das Ergebnis, das störend ist, ist nicht das, wonach ich suche.

Ebenso hat ein freies Spinorfeld

G 2 1 ( P ) = P + Ö ( 1 )
während ein wechselwirkender, nach erster Ordnung in der Störungstheorie, hat
G 2 1 ( P ) = C P + Ö ( Protokoll P 2 )
wie vorher.

Schließlich hat man ein freies massives Vektorfeld

G 2 1 ( P ) = Ö ( 1 )
während präturbative Interaktionen wie üblich Protokolle einführen. Es erscheint mir natürlich zu erwarten, dass das führende Verhalten ohne Störung durch die freie Theorie gegeben ist (die hat G 2 = P 2 ( S 1 ) für Spin S ), aber ich würde gerne etwas über das Sub-Leading-Verhalten in einer nicht störenden Umgebung wissen.

Update: Einheitlichkeit

Benutzer Andrew hat vorgeschlagen, dass man das optische Theorem verwenden kann, um der Abnahmerate der Zweipunktfunktion Grenzen zu setzen: zum Beispiel im Fall eines skalaren Felds, das wir haben

G 2 1 ( P 2 ) P C P 2
für einige konstant C (siehe Andrews Link in den Kommentaren für die Quelle).

Ich bin mir nicht sicher, ob dies als asymptotisch für gilt G 2 weil es nicht auf die Eigenschaften von angewiesen ist G 2 ( X ) (noch ϕ ( X ) ), aber es ist nur eine Folge von S S = 1 . Mit anderen Worten, wir verwenden nicht wirklich die Axiomatik der Felder, sondern die physikalische Forderung einer Einheit S Matrix. Soweit ich weiß, gibt es in AQFT wenig Hinweise auf Einheitlichkeit. Vielleicht verlange ich zu viel, aber ich habe das Gefühl, dass man viel über das sagen kann N -Punktfunktion der Theorie mit nur wenigen Axiomen, à la Wightman.

Tatsächlich glaube ich, dass es möglich ist, den Satz von Froissart zu verwenden , um engere Grenzen für den Zerfall der Zweipunktfunktionen zu erhalten, Grenzen, die restriktiver sind als die des optischen Satzes allein. Aber ich habe diese Alternative aus den gleichen Gründen wie oben nicht im Detail untersucht.

Nach meinem Verständnis (was falsch sein könnte) ist eine Theorie, die den Wightman-Axiomen gehorcht, "bereits renormiert", dh Sie haben keine Vorstellung von "nackten Verbreitern" darin. Wie definieren Sie also die Selbstenergie in einer Wightman-Theorie? Die Selbstenergie ist ein "Störobjekt".
@ACuriousMind gibt es nicht-störende Definitionen irreduzibler (auch bekannt als vollständig verbundener) Korrelationsfunktionen (erhalten durch funktionale Ableitungen der Legendre-Transformation der Partitionsfunktion). In der Praxis ist die irreduzible Zweipunktfunktion nur die Umkehrung der vollständigen Zweipunktfunktion: Π ( P ) = G 2 ( P ) 1 , Wo G 2 ( X ) = ϕ ( X ) ϕ ( 0 ) . Mit anderen Worten, und um es klar zu sagen: Ich frage nach dem Verhalten der Zweipunktfunktion, im Impulsraum, bei P .
@ved danke, aber dieser Beitrag diskutiert den Propagator (dh die kostenlose Korrelationsfunktion). Was ich wissen möchte, ist das Verhalten der interagierenden Korrelationsfunktion.
Interaktionsterme modifizieren grundsätzlich den Massenterm des Propagators, und der Propagator wird eine physische Masse für das Regularisierungsschema auf der Schale (oder in gewissem Maßstab für andere) beinhalten, also wenn Sie es in Betracht ziehen P Grenze, dann würde das Verhalten des Propagators gleich bleiben.
Für das Kopfgeld: Ich suche nach der Asymptotik von Zweipunktfunktionen in einer nicht störungsfreien Umgebung. Danke!
Ich bin mir nicht sicher, ob Sie das im Sinn hatten, aber es gibt eine nicht störungsfreie Grenze, die sich aus der Einheitlichkeit ergibt, über die der Propagator nicht schneller abfallen kann 1 / P 2 als P (Siehe zum Beispiel Gleichungen 84 und 85 der Notizen von meinem Matt Schwartz isites.harvard.edu/fs/docs/icb.topic1146665.files/… ). Er zeigt es für Spin 0, aber ich glaube, dass dieses Ergebnis für allgemeine Spins gilt.
@andre danke! Ich habe die Frage bearbeitet, um Ihren Kommentar zu diskutieren.
@Andrew Die Einheitlichkeit fällt ab 1 / P 2 Das, was Sie zitieren, gilt eigentlich nur für Korrelationsfunktionen, von denen angenommen wird, dass sie im Unendlichen verschwinden. Es gibt vollkommen gesunde CFT wo ϕ ist ein Primäroperator mit Dimension Δ > 2 wo dies nicht der Fall ist. Man muss die sogenannten Subtraktionen durchführen, um die Fourier-Transformationen durchzuführen, und dies impliziert das Vorhandensein eines endlichen Polynoms im Propagator, zusätzlich zu dem abnehmenden Beitrag, wenn P . Versuchen Sie einfach, die 2pt-Funktion eines Feldes Fourier-transformieren ϕ mit Abmessung Δ > 2 , und Sie sehen den Punkt.
Abgesehen von der Einheitlichkeit, siehe meinen obigen Kommentar zu Andrew, „erfordert“ man jedoch eine polynomiale Beschränktheit, die aus der Natur der gemäßigten Verteilung der Wightman-Funktionen stammt. Es wird angenommen, dass die Stringtheorie und andere sehr eigenartige Theorien (wie das Galileon) diese Bedingung zu verletzen scheinen, da sie ein gewisses Maß an Nicht-Lokalität eingebaut haben. Was Ihren letzten Kommentar zur Froissart-Grenze betrifft, so gibt es viele interessante und gut definierte Theorien (z. B. alle lückenlosen, CFTs, ​​Schwerkraft usw.), bei denen sie verletzt wird.
@TwoBs huh, das ist wirklich interessant und daran hatte ich nicht gedacht. Vielleicht ist die Frage komplizierter als ich dachte, und ich sollte sie ein wenig eingrenzen? Wie auch immer, du hast mir ein paar Themen zum Nachdenken gegeben, danke!

Antworten (1)

Tolle Frage, OP! Ich habe noch keine endgültige Antwort, aber in Ermangelung einer besseren möchte ich erwähnen, dass das Buch Quantenfelder und -strings von Deligne P., Kazhdan D. und Etingof P. die Asymtotik von Wightman-Funktionen bei mehreren Gelegenheiten untersucht . Der vielleicht offensichtlichste ist Abschnitt 1.6 Asymptotik der Wightman-Funktionen (Seite 384), wo wir nachlesen können

W 2 ( X 2 ) X 2 exp [ M X 2 ]
Wo W 2 Ist G 2 im OP und M der kleinste Eigenwert von ist H . Sie scheinen nicht zu erwähnen, wie sich dies auf Theorien mit höherem Spin verallgemeinern lässt. Vielleicht reicht dieses Ergebnis für Ihre Zwecke aus. Lass es mich wissen, bitte.